The limit as x approaches 1 of x / ln (x)

  • Thread starter carlodelmundo
  • Start date
  • Tags
    Limit Ln
In summary: He was simply correcting a mistake and providing further explanation. In summary, the conversation discusses the limit of x --> 1 of (x / ln(x)) and how it results in a semi-indeterminate form. The use of L'Hopital's Rule and algebraic manipulation are mentioned as possible approaches, but it is concluded that the limit does not exist due to the numerator getting closer to 1 while the denominator approaches 0. The idea of removing the natural logarithm is also discussed, but it is determined to be incorrect and the limits are shown to be infinite and negative infinite.
  • #1
carlodelmundo
133
0

Homework Statement



Hi. I'm having problems with the limit of x --> 1 of ( x / ln(x) ).

Homework Equations



L'Hopital's Rule
Algebraic Manipulation

The Attempt at a Solution



I understand that the the limit will give me a semi-indeterminate form (that is, it's answer is 1 / 0).

What I don't understand is how I can manipulate ln (x) so it's not in the denominator. I tried thinking of ways to use e^x... but realized that multiplying by e^x does nothing. I'm presuming that we can raise both the numerator and the denominator by e^x but was not sure if it was legitimate.

Thanks in advance.
 
Physics news on Phys.org
  • #2
If you are getting the 'semi-indefinite form' 1/0, you can forget about manipulating it further. The limit doesn't exist.
 
  • #3
Any time you have a ratio of two continuous functions, and just putting the value gives 1/0 (or more generally any non-zero value over 0) the limit does not exist. As x gets closer and closer to the "target value" the numerator stays close to 1 while the denominator gets close to 0. I.e. the fraction gets larger and larger. There is no limit.
 
  • #4
Thanks, both. I understand it now. I made a numerical table to estimate the limit... and as you two pointed out... the limit does not exist (infinite discontinuities).
 
  • #5
well i don't think so maybe I'm wrong but let's see

lim as x->1 (x/lnx) now me remove the natural lag

lim as x->1 ( e^x/ x) so as X approaches 1 we get lim x->1 (e^1/1)=e :)

and yes it's legit :)
 
Last edited:
  • #6
HallsofIvy said:
Any time you have a ratio of two continuous functions, and just putting the value gives 1/0 (or more generally any non-zero value over 0) the limit does not exist. As x gets closer and closer to the "target value" the numerator stays close to 1 while the denominator gets close to 0. I.e. the fraction gets larger and larger. There is no limit.

To add to this, the limit could be infinity, -infinity, or not exist. If you have something like [tex]\lim_{x\to 0}\frac{1}{x}[/tex], x could approach 0 from the left, from the right, or from both directions. In this case, we can't put a definitive answer.

Note: This only works if you include the extended real line. If not, then it wouldn't exist at all.
 
Last edited:
  • #7
icefirez said:
well i don't think so maybe I'm wrong but let's see

lim as x->1 (x/lnx) now me remove the natural lag

lim as x->1 ( e^x/ x) so as X approaches 1 we get lim x->1 (e^1/1)=e :)
What makes you think that you can remove the natural "lag" (sic) in this way?

As was already said, this limit does not exist. The two one-sided limits are as far apart as they could possibly be.
[tex]\lim_{x \to 1^+}\frac{x}{ln(x)} = \infty[/tex]
while
[tex]\lim_{x \to 1^-}\frac{x}{ln(x)} = -\infty[/tex]

icefirez said:
and yes it's legit :)
 
  • #8
icefirez said:
well i don't think so maybe I'm wrong but let's see

lim as x->1 (x/lnx) now me remove the natural lag

lim as x->1 ( e^x/ x) so as X approaches 1 we get lim x->1 (e^1/1)=e :)

and yes it's legit :)
Actually ex/lnx cannot be simplified unlike elnx = x. For your second expression you'll get:

[tex]
\lim_{x \to 1^+}e^(x/lnx) = \infty
[/tex]
[tex]
\lim_{x \to 1^-}e^(x/lnx) = 0
[/tex]
 
  • #9
Mark44 said:
What makes you think that you can remove the natural "lag" (sic) in this way?

As was already said, this limit does not exist. The two one-sided limits are as far apart as they could possibly be.
[tex]\lim_{x \to 1^+}\frac{x}{ln(x)} = \infty[/tex]
while
[tex]\lim_{x \to 1^-}\frac{x}{ln(x)} = -\infty[/tex]


sorry yes I'm wrong... but you don't have to be rude and I know that you have to write log instead of "LAG" but please it's not the end of world.
 
  • #10
There was nothing rude about Mark44's response.
 

1. What is the value of the limit as x approaches 1 of x / ln(x)?

The value of this limit is 1.

2. How do you evaluate the limit as x approaches 1 of x / ln(x)?

To evaluate this limit, you can use L'Hôpital's rule or rewrite the expression as ln(x) / (1/x) and use the limit definition of the derivative.

3. Why is the limit as x approaches 1 of x / ln(x) important?

This limit is important because it is a fundamental concept in calculus and is used to calculate the growth rate of a function as it approaches a certain value.

4. Can the limit as x approaches 1 of x / ln(x) be computed by plugging in x = 1?

No, you cannot simply plug in x = 1 to evaluate this limit. This will result in an undefined expression, known as an indeterminate form.

5. How does the function x / ln(x) behave as x gets closer to 1?

As x approaches 1, the function x / ln(x) approaches 1 from the left and from the right. This means that the function has a horizontal asymptote at y = 1 and the graph of the function gets closer and closer to this line as x approaches 1.

Similar threads

  • Calculus and Beyond Homework Help
Replies
34
Views
2K
  • Calculus and Beyond Homework Help
Replies
8
Views
920
  • Calculus and Beyond Homework Help
Replies
8
Views
665
  • Calculus and Beyond Homework Help
Replies
14
Views
1K
  • Calculus and Beyond Homework Help
Replies
5
Views
807
  • Calculus and Beyond Homework Help
Replies
2
Views
3K
  • Calculus and Beyond Homework Help
Replies
3
Views
6K
  • Calculus and Beyond Homework Help
Replies
15
Views
1K
  • Calculus and Beyond Homework Help
Replies
5
Views
909
  • Calculus and Beyond Homework Help
Replies
5
Views
1K
Back
Top